Math is fun!

Algebra Level 3


The answer is 36.

This section requires Javascript.
You are seeing this because something didn't load right. We suggest you, (a) try refreshing the page, (b) enabling javascript if it is disabled on your browser and, finally, (c) loading the non-javascript version of this page . We're sorry about the hassle.

3 solutions

I actually studied this problem just a year ago, twice, and my head exploded, twice.

x 2 t 1 2 + y 2 t 3 2 + z 2 t 5 2 + w 2 t 7 2 1 = 0 \displaystyle \frac{x^{2}}{t-1^{2}} + \frac{y^{2}}{t-3^{2}} + \frac{z^{2}}{t-5^{2}} + \frac{w^{2}}{t-7^{2}} - 1 = 0 for t = 2 2 , 4 2 , 6 2 , 8 2 t = 2^{2}, 4^{2}, 6^{2}, 8^{2} .

Multiply ( t 1 ) ( t 9 ) ( t 25 ) ( t 49 ) (t-1)(t-9)(t-25)(t-49) both sides.

( t 9 ) ( t 25 ) ( t 49 ) x 2 + ( t 1 ) ( t 25 ) ( t 49 ) y 2 + ( t 1 ) ( t 9 ) ( t 49 ) z 2 + ( t 1 ) ( t 9 ) ( t 25 ) w 2 ( t 1 ) ( t 9 ) ( t 25 ) ( t 49 ) = 0 (t-9)(t-25)(t-49)x^{2} + (t-1)(t-25)(t-49)y^{2} + (t-1)(t-9)(t-49)z^{2} + (t-1)(t-9)(t-25)w^{2} - (t-1)(t-9)(t-25)(t-49) = 0 (1)

Let P ( t ) = ( t 1 ) ( t 4 ) ( t 9 ) ( t 25 ) ( t 9 ) ( t 25 ) ( t 49 ) x 2 ( t 1 ) ( t 25 ) ( t 49 ) y 2 ( t 1 ) ( t 9 ) ( t 49 ) z 2 ( t 1 ) ( t 9 ) ( t 25 ) w 2 P(t) = (t-1)(t-4)(t-9)(t-25) - (t-9)(t-25)(t-49)x^{2} - (t-1)(t-25)(t-49)y^{2} - (t-1)(t-9)(t-49)z^{2} - (t-1)(t-9)(t-25)w^{2}

We know that t = 2 2 , 4 2 , 6 2 , 8 2 t = 2^{2}, 4^{2}, 6^{2}, 8^{2} are the roots of P ( t ) P(t) and deg ( P ) = 4 \deg(P) = 4 .

Therefore, P ( t ) = ( t 2 2 ) ( t 4 2 ) ( t 6 2 ) ( t 8 2 ) P(t) = (t-2^{2})(t-4^{2})(t-6^{2})(t-8^{2}) . (2)

Compare the coefficients of t 3 t^{3} from (1) and (2) we get

x 2 + y 2 + z 2 + w 2 + 1 + 9 + 25 + 49 = 2 2 + 4 2 + 6 2 + 8 2 x^{2}+y^{2}+z^{2}+w^{2}+1+9+25+49 = 2^{2}+4^{2}+6^{2}+8^{2}

Which gives x 2 + y 2 + z 2 + w 2 = 36 x^{2}+y^{2}+z^{2}+w^{2} = \boxed{36} ~~~

@Samuraiwarm Tsunayoshi Perfect solution, did this problem so many time and could not get it, but i now how to do it now.

Mardokay Mosazghi - 6 years, 8 months ago

Log in to reply

I'm not sure if we can use matrices to solve that.

Samuraiwarm Tsunayoshi - 6 years, 8 months ago

Log in to reply

You can (use matrices) to solve for the values of x , y , z , w x, y, z, w , but the point is that is not necessary.

There are several variants of this question, which involve finding a suitable equation and then manipulating it.

Calvin Lin Staff - 6 years, 8 months ago

Log in to reply

@Calvin Lin Yep, I won't use matrices to solve x , y , z , w x,y,z,w individually. I was thinking about getting x 2 + y 2 + z 2 + w 2 x^{2}+y^{2}+z^{2}+w^{2} directly.

Samuraiwarm Tsunayoshi - 6 years, 8 months ago

Log in to reply

@Samuraiwarm Tsunayoshi That's how I would do it if I had 1 hour to waste inverting a 4x4 matrix.

Finn Hulse - 6 years, 8 months ago

Log in to reply

@Finn Hulse ha ha i tried it and takes a lot of time ->so much time wasted.

Mardokay Mosazghi - 6 years, 8 months ago

Log in to reply

@Mardokay Mosazghi There's no such thing as wasting time for math lol.

Samuraiwarm Tsunayoshi - 6 years, 8 months ago

Log in to reply

@Samuraiwarm Tsunayoshi i mean when you have the easier method why waste time especially on contests. But yeah i agree.

Mardokay Mosazghi - 6 years, 8 months ago

how did you come up with co-efficient of (2) equation? .. shouldnt it be

( 2 2 + 4 2 + 6 2 + 8 2 ) -({ 2 }^{ 2 }+{ 4 }^{ 2 }+{ 6 }^{ 2 }+{ 8 }^{ 2 })

Shivam Bhagat - 6 years, 8 months ago

the co-efficient of t^3 in (t-1)(t-2)(t-3)(t-4) is negative and it is being multiplied by -1.. so It should be 1+9+25+49 .... the overall co-efficient should be x 2 + y 2 + z 2 + w 2 + 1 + 2 + 4 + 9 { x }^{ 2 }+{ y }^{ 2 }+{ z }^{ 2 }+{ w }^{ 2 }+1+2+4+9 AND on the right hand side it should be ( 2 2 + 4 2 + 6 2 + 8 2 ) -({ 2 }^{ 2 }+{ 4 }^{ 2 }+{ 6 }^{ 2 }+{ 8 }^{ 2 })

Shivam Bhagat - 6 years, 8 months ago

Log in to reply

Oh god, I made a mistake a little bit. P ( t ) P(t) was supposed to be ( t 1 ) ( t 4 ) ( t 9 ) ( t 25 ) ( t 9 ) ( t 25 ) ( t 49 ) x 2 ( t 1 ) ( t 25 ) ( t 49 ) y 2 ( t 1 ) ( t 9 ) ( t 49 ) z 2 ( t 1 ) ( t 9 ) ( t 25 ) w 2 (t-1)(t-4)(t-9)(t-25) - (t-9)(t-25)(t-49)x^{2} - (t-1)(t-25)(t-49)y^{2} - (t-1)(t-9)(t-49)z^{2} - (t-1)(t-9)(t-25)w^{2} . Thank you for pointing me out!

Samuraiwarm Tsunayoshi - 6 years, 8 months ago

Log in to reply

Taking the previous assumption was not permissible as it resulted in the answer as ( -36 ) which isnt possible.

Shivam Bhagat - 6 years, 8 months ago

Amazing solution this one, but I actually solved it in 2 minutes, maybe by accident. Could someone please point me out where my solution could have gone wrong? This is how it goes: As with the first equation we definitely get a positive number(no matter the 1) and in the last definitely a negative, we can assume that in the first y^2=z^2=w^2=0, and then x^2/(2^2-1^2)=1 So x^2= 4-1=3 In the second equation, accordingly, we say that x^2=z^2=w^2=0, and, accordingly, find out that y^2=7 The same thing with the last two, and there we have 3+7+11+15=36 Not a very nice explanation, I would be very grateful if you explained me why my solution actually worked.

Михаил Зайчик - 5 months, 1 week ago
Rohan Chandra
Oct 1, 2014

this ques took hell of time. after many faults I was finally able to solve this one. Too good question ... !

thnaks Rohan

Mardokay Mosazghi - 6 years, 8 months ago
Sharky Kesa
Nov 8, 2014

@Mardokay Mosazghi !!! I posted this problem before. AIME 1984 !!!

i didn't know sorry.

Mardokay Mosazghi - 6 years, 7 months ago

0 pending reports

×

Problem Loading...

Note Loading...

Set Loading...